The Student Room Group

AQA A2 MFP2 Further Pure 2 – 24th June [Exam Discussion Thread]

Scroll to see replies

Original post by 2014_GCSE
Do you convert tanh^2(x) in a formula such as 1 + tan^2(x) = sec^2(x) because doesn't this technically contain a sinh^2(x)?

Sorry, FP2 is an extra voluntarily module so I've kind of ignored it until now and I forget a bunch of small things like this.


That expression is obtained by dividing cosh^2x - sinh^2x = 1 by sinh^2x. Which gives you a negative 1 rather than positive.
Reply 121
image.jpgWould my layout of method of differences be alright cause the mark schemes don't really specify any certain way
I am so utterly screwed for this.
Original post by Hjyu1
image.jpgWould my layout of method of differences be alright cause the mark schemes don't really specify any certain way


Yeah that's fine, basically the same as they do it in the AQA textbook.
Original post by -jordan-
That expression is obtained by dividing cosh^2x - sinh^2x = 1 by sinh^2x. Which gives you a negative 1 rather than positive.


Is there any list out there that has all of the hyperbolic identities ?
Original post by 2014_GCSE
Is there any list out there that has all of the hyperbolic identities ?


You don't need to know all of these but I found this:
http://www.alcyone.com/max/reference/maths/hyperbolic.html
Original post by JackHKeynes
You don't need to know all of these but I found this:
http://www.alcyone.com/max/reference/maths/hyperbolic.html


Doing God's work. <3
Can some one help me with the proof by induction question in june 2007 please ? THis chapter really confuses me :/
Original post by Roxanne18
Can some one help me with the proof by induction question in june 2007 please ? THis chapter really confuses me :/


Which bit is it that you can't do specifically? You just do the initial step but starting from n=2 rather than n=1 and then assume true for n=k. Work out your goal as usual by subbing in k+1. Then the sum multiplied by (1-1/(k+1)^2) gives you your goal therefore it's proven by induction (which you can just quote from part a) I assume.
Reply 129
Would max / min values of modules of z always be on opposite ends
Original post by Hjyu1
Would max / min values of modules of z always be on opposite ends


Come on, you have to give more information then that, makes no sense the way you've put it.
Reply 131
Original post by IrrationalRoot
Come on, you have to give more information then that, makes no sense the way you've put it.


On a circle *
Original post by -jordan-
Which bit is it that you can't do specifically? You just do the initial step but starting from n=2 rather than n=1 and then assume true for n=k. Work out your goal as usual by subbing in k+1. Then the sum multiplied by (1-1/(k+1)^2) gives you your goal therefore it's proven by induction (which you can just quote from part a) I assume.


Ah right okay, thank you! I'll have another look now (:
Predictions for tomorrow? I reckon hard de Moivre's as last year's was quite easy
(also if anyone knows any resources for hard last minute de Moivre questions it would be appreciated :tongue: )
Original post by sarcastic-sal
Predictions for tomorrow? I reckon hard de Moivre's as last year's was quite easy
(also if anyone knows any resources for hard last minute de Moivre questions it would be appreciated :tongue: )

I will accept my fate as soon as I see 9 questions with hard-core DM question as 9th.
Original post by Hjyu1
On a circle *


Give an example
Original post by C0balt
I will accept my fate as soon as I see 9 questions with hard-core DM question as 9th.


Haha. I hope AQA don't mess this one up, they're too creative recently.
Original post by sarcastic-sal
Predictions for tomorrow? I reckon hard de Moivre's as last year's was quite easy
(also if anyone knows any resources for hard last minute de Moivre questions it would be appreciated :tongue: )


June 2013 DM was lovely, did that paper as a mock very recently.
Original post by C0balt
I will accept my fate as soon as I see 9 questions with hard-core DM question as 9th.


I read this as hard-core death match...

Can't wait till exams are over, my brain is actually fried
Reply 139
What's the expression for alpha^3 + beta^3 + gamma^3?

Quick Reply

Latest